Si Fermat avait la solution.. - Page 3
Répondre à la discussion
Page 3 sur 3 PremièrePremière 3
Affichage des résultats 61 à 83 sur 83

Si Fermat avait la solution..



  1. #61
    leg

    Re : Si Fermat avait la solution..


    ------

    PREAMBULE

    Si l’idée merveilleuse de Fermat était :
    Où choisir un couple de paramètres p et q, formant un triplet de racines carrées : x’ , y’, z’ vérifiant l’égalité

    X + Y = Z ou tel que: x N + yN = z N !

    sous entendu: au départ pour N=1 puis:
    X =x N en changeant l'exposant N= 2, 3 ,4 ,..etc
    Y = yN
    Z = zN


    La formule des triplets pythagoriciens donne un triplet de racines carrées.
    √X ,√Y, √Z ; avec X= x^N ; Y = y^N et Z = z^N .

    Elle donne toutes les solutions entières, X + Y = Z. soit : (p²- q²)² + (2pq)² = (p² + q²)²
    avec p et q, qui parcourent l’ensemble des entiers naturels, dans la puissance N =1.(sous entendu les racines carrées)
    En utilisant la relation du théorème de Pythagore on vérifie par la même une équation de Fermat sans perte de généralité !
    p² - q² =√X , 2pq =√Y, et p²+q² = √Z

    ce qui est le début de mon pdf,
    je ne parle donc que de racines carrées.
    qui s'écrive sous la forme de la formule des triplet pyth, que ces derniers soient des entiers ou des réel algébrique
    c'est à dire: p² - q² =√X , 2pq =√Y, et p²+q² = √Z
    d'accord?

    -----

  2. #62
    Médiat

    Re : Si Fermat avait la solution..

    Ce que j'ai compris :

    Si le triplet de réels (x, y, z) est une solution de xN + yN = zN, pour N un entier non nul quelconque, alors il existe un couple de réels (p, q) tels que
    p² - q² = xN/2, 2pq = yN/2 et p² + q² = zN/2.

    Je suis d'accord et j'attends la suite avec impatience ...
    Je suis Charlie.
    J'affirme péremptoirement que toute affirmation péremptoire est fausse

  3. #63
    leg

    Re : Si Fermat avait la solution..

    dans la suite du pdf que sous entend cette phrase:

    Donc les seules solutions qui nous intéressent sont les primitives avec K = 1 ;
    car bien entendu, ces solutions multiples d’une solution primitive, données par p’ et q’ réels, ne sont pas différentes de celle donnée justement par p et q et un facteur K >1, ce qui n’apporte rien de plus ni une contradiction!


    ceci indique clairement qu'il n'existe pas de triplet primitif dans N =1, paramétré avec p' et q' réels différent de ceux qui sont donné par p et q entiers naturels

    et si mon triplet existe par supposition dans N=2 ou N=3 il s'agit toujours d'un triplet d'entiers, donc paramèttré par p et q
    supposer qu'il existe un triplet paramèttré par p' et q' dans une de ces puissance c'est admettre que le théorème de Fermat est faux!

    c'est pour cette raison que Fermat été obligé de passer par les puissance paires , il n'avait pas les outils mathématique de A Willes ni même ceux de S. Germain pour le cas N=5; de plus cela ne lui aurait servi à rien

    mais par contre, il lui était facile de passer par N pair pour redescendre sur N premier, c'est à dire l'inverse de ce qu'on fait tous les mathématiciens depuis + de 3 siecles et demi.

    soit on démontre l'absence de paraméttrage pour N = 4,puis 6 ce qui nous oblige à passer par N=3 ensuite, et au passage de faire remarquer que pour N= 2 il n'y a pas de couple p' et q' réels paraméttrant une solution primitive dans cette dernière puissance, qui ne serait paraméttré par p et q.

    puis de conclure de Façon générale:

    ce qui est vrai pour N=2,3,4 6 et on peut rajouter 10 et 5 de S. Germain, est forcément vrai pour tout N !

    voila pourquoi Fermat avait la solution.
    ce qui sous entend que si mes raisonnement sont vrai il est bien le premier à avoir démontré son théorème!

    et dans tout les cas il est bien le premiers à l'avoir démontré de façon générale pour N pair >2
    ce qui n'a jamais été fait ....jusqu'a maintenant...

  4. #64
    leg

    Re : Si Fermat avait la solution..

    Citation Envoyé par Médiat Voir le message
    Ce que j'ai compris :

    Si le triplet de réels (x, y, z) est une solution de xN + yN = zN, pour N un entier non nul quelconque, alors il existe un couple de réels (p, q) tels que
    p² - q² = xN/2, 2pq = yN/2 et p² + q² = zN/2.

    Je suis d'accord et j'attends la suite avec impatience ...
    pourquoi tu divise N par 2 ?
    si N=3?
    pour moi ceci indique bien que l'exposant N/2 on est dans les puissances premières car N est pair
    p² - q² = xN/2, 2pq = yN/2 et p² + q² = zN/2.
    ce qui est vrai il existe bien un couple p' et q'

    ("pour rester dans le même paraméttrage il vaut mieux dire que p' et q' réel et non p et q que je désigne comme entier")
    donc tu remarqueras qu'il me faut d'abord résoudre ces deux cas N=4 et N=6

    comprends tu pourquoi ?
    (tu as pratiquement la réponse sur le post au dessus)
    ensuite tu me demanderas de t'expliquer ces deux cas (4 et 6) si tu ne comprends pas mes raisonnements
    A+

  5. #65
    leg

    Re : Si Fermat avait la solution..

    attention Médiat:

    p² - q² = xN/2, 2pq = yN/2 et p² + q² = z N/2.
    ce qui est vrai il existe bien un couple p' et q'si et seulement si ce triplet d'entiers existe paraméttré aussi par p et q entiers!
    le triplet étant des entiers à la puissance N/2 avec N pair >2
    je dis >2 car pour N =1 il existe tous les triplets primitifs paraméttré par p et q entier

    exist'il un triplet primitif paramèttré par p' et q' réel d'apres toi ?

  6. #66
    Médiat

    Re : Si Fermat avait la solution..

    pourquoi tu divise N par 2 ?
    si N=3?
    pour moi ceci indique bien que l'exposant N/2 on est dans les puissances premières car N est pair
    p² - q² = xN/2, 2pq = yN/2 et p² + q² = zN/2.
    ce qui est vrai il existe bien un couple p' et q'
    Je divise par 2 pour prendre la racine, si N=3 cela fait 3/2, cela n'impose pas que N soit pair.
    C'est quoi p' et q', d'où viennent-ils, sont-ce des réels, des entiers, quelles relations vérifient-ils ?
    Je suis Charlie.
    J'affirme péremptoirement que toute affirmation péremptoire est fausse

  7. #67
    leg

    Re : Si Fermat avait la solution..

    Citation Envoyé par Médiat Voir le message
    C'est quoi p' et q', d'où viennent-ils, sont-ce des réels, des entiers, quelles relations vérifient-ils ?
    au départ je défini le couple de paramèttre par p et q entiers
    puis ensuite je passe au couple de paramèttre p' et q' réels
    la raison:
    je suppose l'existence d'un triplet qui me vérifie l'équation x^4 + y^4 = z^4
    les racines carrées : p²-q² =(x²), 2pq = (y²) et
    p²+q² = (z²) donc triplet paraméttré par p et q entiers ok
    je démontre que cette solution n'existe pas; donc le triplet non plus.
    il devient alors évident qu'il ne peut pas exister un ou ce triplet, paraméttré de la même façon par p' et q' réel !
    ou alors ma démonstration serait fausse c'est à dire que le cas n= 4 est faux
    je peux me servir d'un couple de paramèttres p et q entier ou p' et q' réels il n'y a pas de contrainte.

    par exemple p =2 et q=1 donne le triplet 3,4 et 5
    p' et q' donne le triplet 6,8 et 10 soit p' =2 (sqrt de 2) et q = sqrt de 2,
    mais ce couple de paramèttresp' et q', ne m'apporte rien de plus que le couple p =2, q =1 avec un facteur K = 2
    si le triplet 6,8 et 10 était paraméttré uniquement par p' et q' il serait alors un triplet primitif! et la formule des triplets qui donne toutes les solutions entières, serait fausse car il existerait ce triplet primitif qui n'est pas donné uniquement par p et q entier !

    c'est justement ce que l'on va montrer, et pour cela il me faut bien montrer que le cas N= 4 et N=6 n'à pas de solution, pour cela il me faut bien d'abord paraméttrer le triplet supposer exister dans N=2 et 3 comme ces triplets sont constituer d'entiers je me sert d'un couple de paramèttres p et q entiers pour soulever la contradiction dans N=4 et dans N=6.

    est ce que tu comprends l'idée et le raisonnement?
    A toi

  8. #68
    leg

    Re : Si Fermat avait la solution..

    ("si le triplet 6,8 et 10 était paraméttré uniquement par p' et q' il serait alors un triplet primitif! et la formule des triplets qui donne toutes les solutions entières, serait fausse car il existerait ce triplet primitif qui n'est pas donné uniquement par p et q entier !)"

    supprime uniquement désolé

  9. #69
    Médiat

    Re : Si Fermat avait la solution..

    au départ je défini le couple de paramèttre par p et q entiers
    J'essaye de repartir de choses démontrées, par exemple mon post N°62 (il faudrait d'ailleurs ajouter que x, y et z sont des réels strictement positifs pour éviter quelques problèmes).

    Dans ce post p et q sont des réels, maintenant tu dis que ce sont des entiers, je veux bien, mais ce n'est pas démontré !
    Je suis Charlie.
    J'affirme péremptoirement que toute affirmation péremptoire est fausse

  10. #70
    leg

    Re : Si Fermat avait la solution..

    Citation Envoyé par Médiat Voir le message
    J'essaye de repartir de choses démontrées, par exemple mon post N°62 (il faudrait d'ailleurs ajouter que x, y et z sont des réels strictement positifs pour éviter quelques problèmes).

    Dans ce post p et q sont des réels, maintenant tu dis que ce sont des entiers, je veux bien, mais ce n'est pas démontré !
    on est obligé de passer par deux étapes
    tu oublies qu'une solution carrée est avant tout donnée par p et q entiers
    (x²)² +(y²)² =(z²)² : solution carrée dans N = 4 est N=2
    (x3)² +(y3)² =(z3)²:
    solution carrée dans N = 6 est N=2 mais aussi N=3

    une solution entière primitive dans N= 2 et avant tout donnée par p et q entiers et non par p' et q' réel mais supposons qu'une telle solution primitive dans une de ces deux puissances existent comment pourrait on dire que la fomule des triplets pythagoriciens donne toute les solution entières puisque tu admets qu'elle serait donnée uniquement p' et q' réel
    c'est toi qui a dit que p et q sont réels et non entiers moi je t'ai dit que pet q peuvent être entier ou réel = p' et q'
    regarde bien ce que j'ai dit.
    par contre il est évident que si le triplet est constitué de réel algébrique comme racine carrée de 8 =2^3, racine carrée de 27 =3^3, racine carrée de 125 = 5^3 mais là, la solution serra uniquement dans le cas de N=3et non 6 et forcément non plus dans N=2
    donc dans ce cas le couple de paramèttres p' et q' sont effectivement deux réels positifs par exemple:
    tel que p'² -q'²= sqrt 27 ,2p'q' = sqrt 8 et p'² + q'²=sqrt de 27
    ("en supposant que ce triplet est solution de l'équation Fermat pour N=3")
    mais je doit d'abord montrer l'impossibilité dans N=4 et N=6 car dans ces deux cas je peux utiliser le couple de paramèttres p et q entiers du fait que mes racines carrées sont entières et donneraient une solution dans N=2 ainsi que dans N=4 puis 6 et 3 (suivant la forme des racines carrées si ce sont des carrés parfaits ou des cubes)
    le fait que p et q soient deux entiers et avant tout une obligation pour ces deux cas cité.
    A toi

  11. #71
    Médiat

    Re : Si Fermat avait la solution..

    Je reviendrai quand je verrai une démonstration, c'est à dire l'exposé d'hypothèses complètes, un certain nombres de déductions justifiées et des conclusions.

    Si possible restreint à un cas particulier, N = 13 par exemple (en tout cétat de cause, pas un nombre pair), cela devrait simplifier la démonstration.
    Je suis Charlie.
    J'affirme péremptoirement que toute affirmation péremptoire est fausse

  12. #72
    leg

    Re : Si Fermat avait la solution..

    Citation Envoyé par Médiat Voir le message
    Je reviendrai quand je verrai une démonstration, c'est à dire l'exposé d'hypothèses complètes, un certain nombres de déductions justifiées et des conclusions.

    Si possible restreint à un cas particulier, N = 13 par exemple (en tout cétat de cause, pas un nombre pair), cela devrait simplifier la démonstration.
    comme tu veux
    mais je trouve désolant que lorsque l'on parle de deux cas particulier relativement simple N =4 et 6 je fais l'effort de te faire comprendre pourquoi,
    tu ne regardes même pas le cas N=4 et N=6 pourtant justifié,
    tu ne veux pas de puissances paires, ce qui est pour le moins curieux, et qui permèttent d'expliquer ma démarche
    qui plus est, si une solution exister dans une de ces deux puissance il est évident que le théorème de fermat serait faux et qu'en plus je pourrai paraméttrer un triplet pythagoricien de carrés ou de cubes avec un couple de paramèttres p et q entier ce qui ,à l'air de te poser problème.
    ta seule réponse: démontre le cas avec N=13
    moi je vais, pour te faire plaisir le faire avec N= 2N premier - 1 qui est le 45èmè nombre de mersenne quand je l'aurait trouvé

    alors que depuis le début je te donne la raison de passer en premier par N pair ce qui ne porte pas à une restriction et de plus mon pdf passe par là!
    enfin; à nouveau tu oublies que l'on ne peut démontrer de façon générale avec un seul cas y compris N=13 qui a été démontrer sans apporter une solution générale, alors la restriction uniquement à ce cas ou un autre et totalement absurde .
    amicalement

  13. #73
    leg

    Re : Si Fermat avait la solution..

    revenons au début de Ce petit résumé du pdf, (qui devrait vous permettre de comprendre l’idée générale et ce que P de Fermat à du faire,)

    T1 :
    Si X est un carré, alors Z ne peut en être un !

    De T1 , on en tirera T2 : Si Y est un carré, alors Z ne peut en être un !)

    Preuve :
    (« Triplet primitif, où le facteur commun K = 1 »)

    Si p² - q² = x² , 2pq = y² , et p² + q² = z²
    (« équivalent à l’écriture : x² = x4/2 , y² = y4/2 , z² = z4/2 »)

    Comme nous somme dans les puissance paires > 2 , le couple de paramètres : p et q sont tout d’abord deux entiers non nul, devant aussi donner une solution primitive dans N = 2, ce qui ne nuit aucunement à la généralité et au fait qu’il pourrait aussi s’agir de deux réels positifs notés p’ et q’
    A)
    Nous obtenons : p² - q² = x² où p, q est x sont pythagoriques, on pourrait dire : p = z’ et q = y’
    Ainsi que : p² + q² = z² où p , q est z sont aussi pythagoriques, là aussi on pourrait dire : p = x’ et q = y’ (« il vient de suite la contradiction x’ ≠ z »)

    Il existe donc :
    p1 et q1, tel que : 2 p1 q1 = q = y’;
    ainsi que p2 et q2 , tel que : 2 p2 q2 = q = y’

    d’où il vient: p1 = p2 = p3 et q1 = q2 = q3
    c'est-à-dire un couple p3 et q3 qui aurait paramètré ces deux triplets, ce qui est impossible. Il ne peut donc exister une solution de cette forme dans N = 2 et à plus forte raison dans N = 4 pourrait on déjà conclure ! (« là n’est pas le but »)

    B)
    Ceci nous fait remarquer que dans cette supposition on aurait aussi x’ ≠ z alors que: p = x’ ou z mais pas les deux; du fait que p , q et: x ou z ; sont pythagoriques ! ["ce qui sous entend, que p et q ont été choisis obligatoirement dans un triplet inferieur"] (donc contradiction aussi)

    C)
    De part la propriété des triplets pythagoriciens, on a aussi par obligation:
    (p² - q²)² = ((p² + q²) + (2pq)) ((p² + q²) - (2pq))
    Donc on obtient :
    (x²)² = (z² + y²) (z² - y² ) ? (du fait que le triplet est par supposition, constitué de 3 carrés)

    Ce qui est contradictoire, on se retrouve dans le même cas que A) avec contradiction B) !
    Car on pourrait dans ce cas, remplacer z et y par : p4 et q4 , tel que (p4)² + (q4)² = u² et (p4)² - (q4)² = v² (« avec une descente infinie d’entiers, c’est à dires de racines carrées entières! ») On en conclu que T2 est vrai !

    Quel raisonnement R on en tire ?
    R1)
    Tout simplement, « l’idée de Fermat.. » En effet, si cela existe, alors par obligation il existe deux couples de réels, p’ et q’ ainsi que p’’ et q’’, qui ont paramètré les deux triplets de A),
    A savoir (p,q , x ) avec (p,q, z) ; mais il existe aussi deux autres couples de réels p’’’ et q’’’ ainsi que p’’’’ et q’’’’ ; qui ont paramétré les deux solutions : (z² + y²) = u² et (z² - y² ) = v² du fait que (u² v²) = (x²)² , c'est-à-dire les deux triplets primitifs z,y et u ainsi que z,y et v par supposition ! du fait que ceci ne pourrait être paramètré par des entiers p et q le contraire serait absurde comme on vient de le voir :
    contradiction A) B) et C) !

    R2)
    Si c’est 4 couples de réels existent, alors le couple d’entiers p et q , donnant le triplet primitif :
    x² = x4/2 , y² = y4/2 z² = z4/2 existe aussi ! ce qui est complètement idiot!

    (z² + y²) (z² - y² ) ≠ (x²)² , le produit de deux carrés parfaits premiers entre eux deux à deux ne peut être un carré !
    Ou alors le cas N = 4 est faux, et ses conséquences.
    On en conclu qu’un couple de réel p’ et q’ ne peut paramétrer un triplet Pyth primitif, qui ne serait également donné par p et q entiers, tel que défini dans le pdf,
    Par conséquent, p’ et q’ réels, ne donne un triplet primitif dans les puissances N =1,2 et 4 pas de solution pour N = 4
    (« et le contraire implique que la formule des triplets pythagoricien ne donne pas toutes les solutions entières, ce que l’on saurait depuis des siècles »)

    est ce que dans cette première partie il y a une érreur de raisonnement qui invalide cette demo?

  14. #74
    leg

    Re : Si Fermat avait la solution..

    petite rectif
    x’ ≠ z lire z' et non z
    p = x’ ou z, idem z' au lieu de z
    pareil
    on aurait aussi x’ ≠ z' alors que: p = x’ ou z'

  15. #75
    invite8ef897e4

    Re : Si Fermat avait la solution..

    Bonjour,

    je n'ai pas du tout le temps de lire tout ca meme si j'aimerais bien. J'ai juste une petite question pour leg : pourquoi supposes-tu que Fermat avait vraiment une preuve valide ? Il avait probablement remarque que la conjecture est vraissemblable, et puis il a tout simplement fait une erreur dans une demonstration trop "poilue" pour etre publiee

  16. #76
    leg

    Re : Si Fermat avait la solution..

    Citation Envoyé par humanino Voir le message
    Bonjour,

    je n'ai pas du tout le temps de lire tout ca meme si j'aimerais bien. J'ai juste une petite question pour leg : pourquoi supposes-tu que Fermat avait vraiment une preuve valide ? Il avait probablement remarque que la conjecture est vraissemblable, et puis il a tout simplement fait une erreur dans une demonstration trop "poilue" pour etre publiee
    bonjour humanino
    Cette hypothèse n'est pas très "vraisemblable" car il lui était très facile de généraliser le cas N pair > 2, tel que je le montre ci dessus avec le cas N=4, il est très trivial de réitérer cette démo; tout simplement du fait que toutes puissances N paires implique une solution dans N=2 et tout le monde sait qu'une solution primitive, dans N=2 se paramètre avec p et q entiers tel qu'ils sont définis.

    des lors il ne lui reste que le cas N=3 à résoudre soit par la méthode de L Euler, soit par un raisonnement par l'absurde ce qui je pense était facilement à sa porté.

    Voilà aussi pourquoi, il était obligé de passer par le cas générale N pair avec les moyens de l'époque : la formule des triplets pythagoricien qui démontre de façon très élémentaire le cas N pair.

    Je suppose qu'il généraliser ensuite par ce que vous appeler: une démo par induction ("si cela existait à son époque")
    Ce qui est vrai pour N=1, 2, 4,3 et 6 est vrai pour tout N

    En ce qui me concerne, c’est pour l’histoire;
    personne ne s’est penché sérieusement sur le cas N pair du simple fait, que le raisonnement tenu : si on démontre N = 3 on démontre N = 6 si on démontre N = 13 on démontre N = 26..etc, oui mais cas par cas, et: dans cette regrettable idée, on a oublié de généraliser le cas N pair qui permettait de s’arrêter au cas N = 3, ce que Fermat à fait.

    Donc avant de dire que Fermat s'était peut être trompé, que l'on retrouve au minimum ses arguments c'est à dire:
    1) ses moyens
    2) ses raisonnements
    3) démontrer le cas N pair en se servant de sa démo du cas N=4 et là :
    c'est la sienne que je veux, et non pas celle publiée sur tous les sites Internet, si X est un carré alors Y ne peut en être un avec des lemmes et des contrainte qui n'existait pas à son époque (voir la page 7 et 8 du pdf)
    de plus, il n'existait pas non plus, la rigueur demandée actuellement.
    il pouvait démontrer ce cas précis X et Y, ainsi que celle de la surface carrée d'un triangle rectangle est impossible mais avec ses moyens.
    Et si j'avais eu la chance d'écrire le latin et le vieux français de 1640, j'aurai utiliser ce langage pour montrer sa méthode, et ce qui à pu être ses raisonnement.

    Ce qui m'importe c'est: est il le premier à l'avoir démontré.

    pour tout N pair oui! sans contèste.

    est ce que cela te convient
    regarde le cas N=4 et les raisonnements, ci dessus le dernier post.
    Amicalement leg

  17. #77
    leg

    Re : Si Fermat avait la solution..

    bonjour
    je pourrai presque dire qui ne dit rien consent, pas vrai Martini

    (aprés 2 et 4)

    ***********************

    (« je passe sur le fait que je pourrai choisir p et q dans les cubes ou les racines carrées tel que : p3/2 et q3/2 qui mis au carré me donnerait la contradiction A) et B) tel que (p3/2)² - (q3/2)² = x3 avec (p3/2)² + (q 3/2)² = z3 ainsi que 2 p3/2 q3/2 = y 3 dans ce cas précis, si p et q sont deux cubes , y^3 ne peut exister ; ce qui serait contraire à la
    supposition, ceci implique que p serait un cube, et q un demi cube pair ce qui donnerait bien 2 p^3(q^3/2) = y^3 »)
    n’oublions pas que toutes solutions données dans N=2, par le couple de paramètres p et q , ces derniers sont choisis dans les racines carrées entières !
    Fermat à bien précisé dans une lettre à Carcavi et Huygens qu’il avait utilisé sa descente infinie pour le cas N=3.
    Je suppose que dans ce cas, il avait démontré, que la somme de ces deux nombres (a^3)2 et (2 b^3 c^3) ne peut jamais donné un carré parfait, c'est-à-dire un cube au carré, et un demi cube ! Puisqu’il utilisait les nombres fractionnaires, sinon le cas N= 6 serait vrai ; ce qui nous donnerait bien la relation pythagorique un carré de côté A = c^3 + b^3 avec à l’intérieur un carré de Côté a^3, qui vérifie le théorème de Pythagore tel que : (a^3)2 = (c^3)² + (b^3)² et l’équation de Fermat pour N= 6 et 3, ceci à du lui donner, une descente infinie de racines carrées de cubes ce qui est impossible.

    Donc venons en au Cas N=6:

    T3
    Si X est un cube, alors Z ne peut être un cube, de T3 on en retirera T4 : si Z est un cube alors Y ne peut un cube ou un demi cube
    Si p² - q² = x3 , 2pq = y^3 et p² + q² = z^3 ceci nous ramène à la contradiction C) du cas N = 4
    (« avec : x^3 = x^6/2 ; y^3 = y^6/2 ; z^3 = z^6/2 »)

    C1)
    (p² - q²)² = ((p² + q²) + (2pq)) ((p² + q²) - (2pq)), soit :

    (x^3)² = (z^3 + y^3) (z^3 – y^3 ) on retrouve la contradiction du cas N = 4, deux cubes par addition est soustraction !
    (« p1 et q1, tel que : 2 p1 q1 = y^3; ainsi que p2 et q2 , tel que : 2 p2 q2 = y^3; d’où p1 = p2 = p3 et q1 = q2 = q3 ..etc. qu’elle serait la nature de ces racines carrées qui doivent donner un carré au cube par addition et soustraction ? cela pourrait être des racines carrées entières ou algébriques, mais : (x^3)² est obligatoirement un carré parfait , il doit être solution dans N=2 et 6 et bien sur N=3.»)
    par ex : p = sqrt9 ; q = sqrt2 on obtient bien x² = 49 = (p² - q²)² et il est clair que y ne peut être un entier pour ce cas.


    R3)
    Il existe alors deux couples de réels p’ et q’ avec p’’ et q’’ qui ont paramétré ces deux solutions (z^3 + y^3) = (u²)^3 ; (z^3 – y^3 ) = (v²)^3 car : (x^3)² = ((u²)^3 (v²)^3)
    Donc il existe bien un couple d’entiers p et q choisis ou non dans les racines carrées des cubes ayant donné ce triplet primitif de cubes ce qui est absurde,
    Et il existerait aussi deux triplets primitifs dans les racines carrées entières ou non, correspondant à ces deux solutions, tel que z^3/2 ,y^3/2 , (u²)^3/2 avec z^3/2 ,y^3/2 , (v²)^3/2 , paramètré avec un couple de racines carrées de cube plus petites par ex : z^3/2 et y^3/2
    Ce qui est impossible et contradictoire 2 z^3/2 y^3/2 ne serait pas égale à w^3/2 ! Ce qui entraîne que : (v^3)² = (u^3)² + w^3, n'existerait pas par supposition ; puisque (w^3/2)² = w^3

    R4) : ceci ne serait donc faisable, qu’avec des réels non algébriques.
    Alors :
    (« Euler s’est trompé, sa démonstration est fausse, ainsi que A Willes pour la deuxième fois ainsi que Fermat qui n’aurait pas non plus démontré le cas N=6 »)
    Par conséquent p’ et q’ réels, ne peuvent donner un triplet primitif dans une puissance N = 1,2,3,4 et 6 qui ne serait donné par p et q entiers, ou racines carrées algébriques de produits cubiques tel que de p^3/2 et q^3/2 ; c’est à dire deux racines carrées de cubes, non carrés !
    Je pense qu’il est inutile de pousser jusqu’à S. Germain pour N = 5 ce qui est vrai pour N = 1,2 ,3.. ceci est vrai pour tout N. N +1 , N+2, il n'existe pas un couple de réels paramétrant un triplet primitif, avec solution dans une puissance N premièr.
    On peut considérer, puisque par convention : 1 n’est pas un nombre premiers, alors 1 n’est pas une puissance première !
    Voila pour cette deuxième partie

  18. #78
    martini_bird

    Re : Si Fermat avait la solution..

    Salut,

    Citation Envoyé par leg Voir le message
    je pourrai presque dire qui ne dit rien consent, pas vrai Martini
    Je pars quelques jours, mais promis dès que je rentre, je poste un commentaire.

    Cordialement.
    « Angle éternel, la terre et le ciel, pour bissectrice, le vent. » Garcia Lorca

  19. #79
    leg

    Re : Si Fermat avait la solution..

    Citation Envoyé par martini_bird Voir le message
    Salut,

    Je pars quelques jours, mais promis dès que je rentre, je poste un commentaire.

    Cordialement.
    passe de bonnes vacances Amicalement leg

  20. #80
    leg

    Re : Si Fermat avait la solution..

    bonsoir
    pour relancer un peu le sujet,
    suite aussi à la question posée par humanino:

    fermat aurait'il pu démontrer sous cette forme le cas N=3?

    Cas n=3 démonstration, référence : FERMAT serge.mehl.free.fr/chrono/Fermat.html

    Il s'agit de prouver l'impossibilité d'obtenir, en nombres entiers non nuls, l'égalité x3 + y3 = z3, cas particulier, lorsque n = 3, du célèbre grand et dernier théorème de Fermat

    On se place désormais dans Z, ensemble des entiers relatifs, afin de pouvoir "jouer" sur la symétrie du problème. Si notre égalité ne peut avoir lieu dans Z, elle sera, a fortiori, impossible dans N. Quand on parlera de diviseur, il s'agira toujours, par définition, d'un entier naturel non nul.

    1. Quitte à diviser par leur pgcd, nous pouvons supposer que x , y et z sont premiers entre eux.

    2. Eu égard à l'hypothèse 1 ci-dessus, un seul des trois entiers x, y et z est pair.

    3. Eu égard à la condition 2 et au fait que nous travaillons dans Z, on peut supposer que l'entier pair est z.
    En effet : si x (resp. y) est pair, on change les signes de x et z et on échange leurs rôles.

    L'équation x3 + y3 = z3 peut s'écrire :
    z3 = (x + y)(x² - xy + y²) = (x + y)[(x - y)² + xy]
    Posons x = u + v et y = u - v. Ceci est loisible car x + y et x - y sont pairs : on a x + y = 2u et x - y = 2v. Notre égalité s'écrit :
    z3 = 2u(u² + 3v²)

    Comme x et y sont premiers entre eux, il en est de même des entiers u et v et ces nombres sont de parités différentes. Le nombre u²+ 3v² est alors impair et z étant pair, z3 est divisible par 8. Ainsi, u est un multiple de 4. Posons u = 4t; u est donc pair et v impair et :

    z3 = 8t(16t² + 3v²)
    Il suit que t(16t² + 3v²) est un cube.

    ****************************
    Note : et en référence aux derniers post:

    « Nous avons déjà à ce stade de la démonstration, la même contradiction que le cas N = 4, 6:

    (z^3 + y^3) (z^3 - y^3) = (x²)^3 = (u²)^3 (v²)^3 , A savoir z = x

    En effet : pour que cette équation z^3 = 8t(16t² + 3v²) existe, ainsi que :
    t(16t² + 3v²) est un cube, il faut que :

    (x + y = 2u) = (x - y = 2v), c'est-à-dire que v = u ; ce qui est absurde!
    "on retrouve le même résultat".

    Si t = 1 , u = 4t et v = 4t et z3 = 8 4^3
    Si t = 3 , u = 4t et v = 4t et z3 = 8 12^3
    Si t = 5 , u = 4t et v = 4t et z3 = 8 20^3

    Etc ..etc ; Fermat pouvait il démontrer sous cette forme le cas N=3 ? »
    ****************************** **
    vous pouvez voir la suite sur le site en référence.

    par contre je ne sais pas si il s'agit exactement de la démonstration de L Euler pour N=3 , si quelqu'un pouvait me donner la référence d'un site ou la mettre en pdf. je l'en remerci

  21. #81
    leg

    Re : Si Fermat avait la solution..

    bonjour Martini
    je passe sur le cas N=4

    je remet un peu plus d'explication afin de lever toutes idées, sur l'impossibilité de représenter un triplet de racines carrées dans une puissance première ou son double, vérifiant l'équation de Fermat.

    Formule des triplets pythagoriciens démonstration
    (par averse, les Mathématiques.net)

    (1)
    On prend : a , b, c, apartient aux entiers naturel
    En fait, on peut se ramener au cas où a , b et c sont premiers entre eux.
    On peut montrer aussi que l’on a nécessairement a et b de parité différente.
    Ce qui fait qu'on peut supposer a pair.

    On a : a² = (c + b) (c – b) . Avec : b et c ayant même parité (impair), = c + b et c – b sont pairs et on Ecrit :

    c + b = 2 u et c – b= 2v Donc : a² = 4 u v . Mais b et c , sont premiers entre eux. Donc u et v aussi ; ce qui implique que : u et v sont des carrés. (« si le produit de deus entiers premiers entre eux est un carré, ces deux nombres sont eux même deux carrés ») On écrit alors : c + b = 2n² et. c – b = 2m².
    On a donc : a = 2nm ; b = n² - m² et c = n² + m².

    ****************************** ************

    (2)
    Si maintenant on suppose un triplet de cubes vérifiant le théorème de Pythagore, tel que : il existe un carré de côté A et un carré de côté z^3
    à l’intérieur de A ; nous avons alors :
    A = x^3 +y^3 , ce qui donne
    A² = (2 x^3 y^3) + (x^3)² + (y^3)² = (2 x^3 y^3 ) + (z^3)²
    c'est-à-dire: (x^3)² + (y^3)² = (z^3)²

    (3)
    on suppose y pair, ce qui correspond à l’égalité (1)
    (y^3)² = (z^3 + x^3) (z^3 - x^3) = 4 u v ;
    mais aussi l’égalité 2u 2v avec u un cube au carré =
    (t^3)², et v est ½ cube au carré .
    (ou son double et carré = 2w^3 = g², et où: w^3 / 8 = (r^3)². )

    soit 2² (t^3)² 2w^3 = 4 u v = (y^3)²

    il existe alors un couple d’entier t^3 et g, tel que 2 t^3 g = y^3
    par commodité je peux remplacer : t^3 par p , et g , par q
    donc p est un cube p^3 et q un demi cube de sorte que (2 p^3 (q^3 / 2))² = (y^3)²

    ex : q = 27/2 et p = 8

    4 p² q² = (6²)^3 = (y^3)² = 4 u v, bien sur il faudra que q soit pair, afin d’avoir z^3 et x^3 entiers !

    (4)
    Il est tout aussi évident que :
    (x^3)² = (z^3 + y^3) (z^3 - y^3) ces deux nombres sont impairs et premiers entre eux deux à deux ;
    leurs produits étant un cube au carré, ou l’inverse ce qui ne change rien, ces deux nombres en sont de même :
    (m^3)² et (n^3)²

    (x^3)² = (m^3)² (n^3)² .
    Nous avons obtenu deux nouvelles solutions cubiques, par addition et soustraction plus petites, ce qui serra impossible !

    la suite prochain post

  22. #82
    leg

    Re : Si Fermat avait la solution..

    En effet :
    z^3 - y^3 est égale à : (n^3)² ; où z^3 et y^3, sont par supposition paramètrés, par un couple d’entiers p^3 et (q^3 /2) soit 2 p^3 (q^3 /2) = y^3;

    il existe alors un autre couple e et g’ inférieur, soit:
    y^3/2 = 2eg’ , z^3/2 = e² + g² et e² - g’²
    puisque (z^3 + y^3) = (m^3)² ce qui est absurde ;

    de plus il faudrait aussi que m^3 et n^3 soit paramétré de la même façon, c'est-à-dire que m3 = n3 , puisque
    z^3/2 , y^3/2 et m^3 ou n^3,
    feraient parti d’un triplet pythagoricien inférieur et où:
    z^3/2, ne peut être égale à la fois à :
    e² + g’² et e² - g’²;
    quand bien même ce couple de paramètres, serait deux réels positifs non entiers e’ et g’ !

    on retrouve la même contradiction que pour N = 2 est 4, il existerait aussi un couple de réel donnant X² = (z² + y²) (z² - y²) or ceci est impossible !
    il n’existe pas un tel couple de réels donnant un triplet primitif X , y² et z² et encore moins deux couples de réels !

    (5)
    Donc si ces deux solutions existent, c’est qu’elles seraient paramétrées par deux couples de réels : i’ et h’ ainsi que i’’ et h’’ et il existerait alors le couple p^3 et
    (q^3 /2) donnant x^3 , y^3 et z^3

    Alors si c’est le cas, il y en a une infinité dans N = 3 mais surtout il existe alors, une infinité de racines carrées de cubes de plus en plus petites.
    Car il me suffit alors de choisir p et q dans ces racines carrées de cube pour trouver deux cubes par addition et soustraction ;
    puisque dans ce cas il existerait, des couples de réels : i’ et h’ ainsi que i’’ et h’’ donnant des solutions cubiques par addition et soustraction

    Ce qui donnerait :
    (P^3/2)² + (q^3/2)² = z^3 ; (P^3/2)² - (q^3/2)² = x^3 et comme :
    (z^3 + x^3) (z^3 - x^3) = (y^3)² ;
    une infinité de solutions, qui implique des racines carrées de cubes ; de plus en plus petites.

    En effet (P^3/2)² et (q^3/2)² sont deux cubes donnant z^3 et x^3 par addition et soustraction, il existe alors
    (a^3/2)² et (b^3/2)² < (P^3/2)² et (q^3/2)² paramétrant ces deux solutions

    (6)
    D’où , si : q = (1^3) / 2 et p = 2^3 alors 2pq = y^3
    (y^3)² = (z^3 + x^3) (z^3 - x^3)
    il existerait alors deux autres triplets de racines carrée de cubes, inférieur aux racine carrées de 1^3 et 2^3 ce qui est absurde !

    De ce fait la démonstration de L Euler est fausse ainsi que Fermat !

    En démontrant le cas N = 4 et 6, puis N=3 ;
    on démontre qu’il n’existe pas de couple de réel paramétrant un triplet primitif qui ne serait donné par p et q entier ou algébrique dans N =1 ,2, 4 , 6 mais aussi dans N=3
    S.Germain ayant démontrée le cas n=5, il n’existe donc aucun couple de réel p’ et q’ formant un triangle rectangle primitif, ayant pour côtés : x^n/2 ; y^n/2 ; z^n/2 pour n ≥ 3
    ni un triangle rectangle primitif ayant pour côtés x, y et z entiers non nul premiers entre eux, qui ne serait donné par p et q entiers non nul, premiers entre eux , de parité différente

    cette démonstration et par conséquent générale !
    Fermat n'aurait eu aucun mal à trouver ce raisonnement.

  23. #83
    leg

    Re : Si Fermat avait la solution..

    bonjour
    pour faire suite au post #80

    il va s'en dire que j'ai considéré t impair mais si t est pair cela ne change pas la contradiction! v = aussi 4t

    ******************
    de sorte que l'équation : x3 + y3 = z3 peut s'écrire :

    z3 = (x + y)(x² - xy + y²) = (x + y)[(x - y)² + xy]

    Posons x = u + v et y = u - v. Ceci est loisible car x + y et x - y sont pairs
    on a:
    x + y = 2u et x - y = 2v.

    Notre égalité s'écrit :
    z3 = 2u(u² + 3v²)

    *********************
    ce qui est impossible ! car il devient alors évident que:
    u = 4t cela entraine aussi v ,

    du fait que par supposition z3 existe et qu'il est pair!
    comme t pair ou impair, multiplie (u² + 3v²) afin de donner t(u² + 3v²) un cube, qui est égale à (4t)3

    et que: x et y sont premiers entre eux, cela s'applique aussi à u et v,
    donc de par la supposition v est impair !

    alors, comme il faut et qu'il existe : 3 v² = 3 u²

    par conséquent, il existerait aussi
    x + y = 2u qui est égale à : x - y = 2v. ce qui est contradictoire
    donc cette égalité ne peut exister dans Z :
    z3 = 2u(u² + 3v²)
    et à plus forte raison dans N

    était il alors besoin de pousser plus avant cette démonstration, afin d'utiliser le principe de la descente infinie de Fermat

    ce dernier à pourtant dit qu'il avait utilisé sa méthode de descente infinie pour résoudre la cas N=3 ,
    mais il est clair, qu'il a pu de ce fait, découvrir cette contradiction bien avant la fin de sa descente, et la garder en réserve ....

    et si on revient dans les entiers naturels N non nul, et donc :
    aux triplets de racines carrées
    où pour N = 4 et 6 on trouve x = z

    dans les entiers relatifs, on trouve aussi :
    u = v
    c'est à dire x+y = x-y ...!

    On pourra toujours continuer à supposer que P de Fermat n'avait pas la solution , mais alors il va falloir trouver de sérieux arguments.
    et expliquer pourquoi:
    personne pour le cas N = 4, 6 et 3 n'a relevé ces contradictions; qui il me semble sont élémentaires , d'après la définition qui en est faite.

    il n'en demeure pas moins vrai, qu' à nouveau il ne peut donc exister un couple de réels p' et q' paramétrant un triplet de racines carrées tel que :

    x3/2 , y3/2 , z3/2

    où: x , y et z ne sont des carrés parfaits.

Page 3 sur 3 PremièrePremière 3

Discussions similaires

  1. Dingue! Einstein avait encore raison!
    Par invitecb70ab37 dans le forum Physique
    Réponses: 43
    Dernier message: 05/07/2004, 17h39